Q2

 
eagerlawstudent
Thanks Received: 0
Forum Guests
 
Posts: 13
Joined: September 12th, 2010
 
 
 

Q2

by eagerlawstudent Mon Nov 22, 2010 2:09 am

For question #2, I am confused as to how Jill and Manny are able to be a pair stocked in aisle 5. If someone could map out this possibility that would be great!
User avatar
 
ManhattanPrepLSAT1
Thanks Received: 1909
Atticus Finch
Atticus Finch
 
Posts: 2851
Joined: October 07th, 2009
 
 
 

Re: PT34, S4,G1 - Jill, Kurt, Larisa, Manny, and Olga

by ManhattanPrepLSAT1 Wed Nov 24, 2010 2:40 pm

Good news! That's not what Q2 asks you to do. In this game you will never have more than one clerk stocking an aisle. The assignment of clerks to aisles may change from question to question, but each aisle will always be stocked by one clerk.

Now since each clerk can stock at most two aisles and that Olga stocks exactly one aisle, we can determine that each of the other clerks stocks two aisles

J K L M O
2 2 2 2 1

For Q2 we need to use the frames from the set up in the earlier post. We can see that aisle 5 must be stocked by M or else K. Ruling out any answer choice that contains an M or else a K, eliminates every answer choice but (E) - which is the correct answer.

Hope that helps!
 
csunnerberg13
Thanks Received: 24
Elle Woods
Elle Woods
 
Posts: 62
Joined: April 10th, 2013
 
 
 

Re: Q2

by csunnerberg13 Tue Sep 10, 2013 1:21 pm

What's the best strategy for this question if you did not use/make frames?
 
JosephV
Thanks Received: 9
Jackie Chiles
Jackie Chiles
 
Posts: 38
Joined: July 26th, 2017
 
 
 

Re: Q2

by JosephV Mon Jan 08, 2018 7:52 pm

csunnerberg13 Wrote:What's the best strategy for this question if you did not use/make frames?

I am not sure that this is the "best" strategy but it does not depend on using frames per se. It does assume that you have already scribbled down a "hypothetical" arrangement of the clerks. Probably one of the simpler such arrangements is: L K M K M J O L J. From here you see that M can be on aisle #5. Therefore, any answer that contains M in it is wrong.

Thus, (A) ,(C), and (D) are eliminated and you are left with (B) and (E) to check. Notice that both have O in them. Therefore, in order to determine that an answer is correct or wrong, you'll have to check whether either K or L could be on aisle #5.

Check answer choice (B), i.e. can K be in #5? A possible arrangement is L K J M K M O L J.

Check answer choice (C), i.e. can L be in #5? We know that K is in #2. So if L is in #5, that leaves no room for the MKM chunk to go anywhere but in slots #6-8. Since O must be after both Ks, O must go into #9. But O must also be followed by at least one L for which we have no room. Hence, conclude that L cannot be in slot #5.

Therefore, the correct answer to this question is (B).